- Thu Oct 28, 2021 1:43 pm
#91681
simon,
Answer choice (A) is the correct answer, not answer choice (B).
PresidentLSAT,
It's true that the number of beliefs could stay the same, so wouldn't have to shrink. Further, "many" beliefs may represent a threshold still exceeded even as someone reduced their beliefs. So the author is making a mistake in that area, as Jon Denning pointed out earlier in this thread. If there's any answer choice that points out that flaw, I'd pick it.
There's another flaw, as already discussed, that the author assumes that beliefs needed to survive couldn't be wrong. We'll keep that in mind if we see it in an answer choice .
Answer choice (A) talks about that second flaw. The author says, essentially, "We need a lot of beliefs to survive, so correctness of beliefs can't depend on rejecting them." The statisticians' procedure tends to reduce beliefs, and the author thinks that survival requires a certain minimal set of beliefs. In response, I say: so what? Must it be that correct beliefs are good for survival? And that's what answer choice (A) is getting at.
Answer choice (B) cannot possibly be right - the statisticians' procedure explicitly disallows ever adding to one's beliefs. Who could follow their rule and add beliefs? That's not possible.
Answer choice (C) isn't a flaw the author commits at all.
Answer choice (D) you got rid of for a good reason.
Answer choice (E) is not something the author assumes at all.
Robert Carroll